Presentation is loading. Please wait.

Presentation is loading. Please wait.

Anticancer & Immunosupressants Q.. Questions 1-3. For each of the following statements, select the most appropriate drug which is BEST characterized by.

Similar presentations


Presentation on theme: "Anticancer & Immunosupressants Q.. Questions 1-3. For each of the following statements, select the most appropriate drug which is BEST characterized by."— Presentation transcript:

1 Anticancer & Immunosupressants Q.

2 Questions 1-3. For each of the following statements, select the most appropriate drug which is BEST characterized by the statement. Answers may be used more than once. (A)Doxorubicin (B)Cyclophosphamide (C)Vincristine (D)Cisplatin (E)Methotrexate 1.Associated with the highest incidence of severe nausea during chemotherapyD 2.Binds to the catalytic site of Dihydrofolate Reductase (DHFR) and interferes with pyrimidine synthesis E 3.Causes cardiotoxicityA

3 Questions 4-8. For each of the following statements, select the most appropriate drug which is BEST characterized by the statement. Answers may be used more than once. (A)Paclitaxel (B)Trastuzumab (C)Bleomycin (D)Finasteride (E)Tamoxifen 4.Usefulness in breast cancer therapy is due to inhibition of HER2 proteinB 5.An antiandrogen often used to treat symptomatic benign prostatic hyperplasiaD 6.Often results in pulmonary toxicity C 7.Inhibits the depolymerization of microtubules A 8.Isolated from the Pacific Yew tree A

4 9.The following adverse effect is NOT usually associated with cancer chemotherapeutic agents such as the alkylating agents and the antibiotics (A)Alopecia (B)Nausea (C)Impotence (D)Neutropenia (E)Aglutition

5 10. Which one of the following chemotherapeutic gents is specific for the M phase of the cell cycle? (A)Doxorubicin (B)Busulfan (C)Vincristine (D)Topotecan (E)Methotrexate

6 A 48-year-old woman is being treated for breast carcino­ma. Over the past few days, she has been complaining of dysuria and frequency. Laboratory examination reveals the presence of microscopic hematuria. The next day the patient develops gross hematuria. Which of the follow­ing drugs could be used to treat the side effect from the antineoplastic medication taken by this patient? (A) Cyclophosphamide (B) Mitomycin (C) Mesna (D) Tamoxifen (E) Vincristine

7 A 24-year-old man underwent treatment for Hodgkin lymphoma 1 year ago. He presents with increasing dys­pnea and cough. Physical exam is remarkable for rales bilaterally. Arterial blood gases show hypoxia, and bilat­eral pulmonary infiltrates are seen on chest x- ray. Which of the following chemotherapeutic agents most likely produced these side effects? (A) Bleomycin (B) Cyclophosphamide (C) Doxorubicin (D) Etoposide (E) 5-Fluorouracil (F) Streptozocin (G) Vincristine

8 A 33-year-old man receiving chemotherapy for testicu­lar carcinoma develops signs of renal tubular damage. Which of the following drugs is most likely responsible for this nephrotoxicity? (A) Bleomycin (B) Cisplatin (C) Cyclophosphamide (D) Vinblastine (E) Vincristine

9 Questions 11-13. For each of the following statements, select the most appropriate drug which is BEST characterized by the statement. Answers may be used more than once. (A)Tacrolimus (B)Azathioprine (C)Infliximab (D)Palivizumab (E)Daclizumab 11.A monoclonal antibody useful in solid organ transplantation to prevent HVGDE 12.A human monoclonal antibody useful in the treatment of Respiratory Syncytial Virus (RSV) infection in neonatal and pediatric patients D 13.Inhibits the action of FK binding protein and is useful as an adjunct to Cyclosporine A

10 Relevant Slide No-10(Chapter9) 1.An anticancer chemotherapeutic agent that acts by first order kinetics would be expected to kill a a.Constant number of cancer cells b.Constant proportion of cancer cells c.Variable number of cells depending on the drug half life d.Variable number of cells depending on the proportion of cells in S phase e.Variable proportion of cells depending on the number of cells in Go phase

11 Relevant Slide No-15(Chapter9) 1.The tumor that is least susceptible to cell-cycle-specific (CCS) anticancer agents is a. Acute lymphoblastic leukemia b. Acute granulocytic leukemia c. Burkitt’s lymphoma d. Adenocarcinoma of the colon e. Choriocarcinoma

12 Relevant Slide No-15(Chapter9) The tumor that is least susceptible to cell-cycle-specific (CCS) anticancer agents is a. Acute lymphoblastic leukemia b. Acute granulocytic leukemia c. Burkitt’s lymphoma d. Adenocarcinoma of the colon e. Choriocarcinoma

13 Relevant Slide No-16(Chapter9) Which of the following chemotherapeutic agents is specific for the M phase of the cell cycle? A. Cytarabine B. Daunorubicin C. Hydroxyurea D. Mechlorethamine E. Vincristine

14 Relevant Slide No-16(Chapter9) 2.The phase of the cell cycle that is resistant to most chemotherapeutic agents and requires increased dosage to obtain a response is the a.M phase b. G2 phase c. S phase d. G0 phase e. G1 phase

15 Relevant Slide No-16(Chapter9) Of the following, which is not a CCS agent a. Mercaptopurine (6-MP) b. 5-FU c. Bleomycin d. Busulfan e. Vincristine

16 A 49-year-old man develops signs and symptoms of peripheral neuropathy while being treated for multiple myeloma with the M- 2 protocol consisting of carmustine, cyclophosphamide, melphalan, prednisone and vincristine. Which of these drugs is most likely responsible for the peripheral neuropathy in this patient? A.Carmustine B.Cyclophosphamide C.Melphalan D.Prednisone E.Vincristine Answer: E Vicristine neurotoxicity; Vinblastine bone marrow toxicity

17 Relevant Slide No-18(Chapter9) 1.A 67 year old woman is being treated for metastatic ovarian cancer with cisplatin and cyclophosphamide. To prevent nausea and vomiting, she is given an selective 5HT3 blocker. Which drug was she given? Dimenhydrinate Dronabinol Metoclopramide Ondansetron Prochlorperazine

18 Relevant Slide No-30(Chapter9) A nucleophilic attack on deoxyribonucleic acid (DNA) that causes the disruption of base pairing occurs as a result of the administration of a. Cyclophosphamide b. 5-FU c. Methotrexate d. Prednisone e. Thioguanine

19 A 60-year-old man with metastatic colorectal cancer is being treated with 5- fluorouracil plus leucovorin and a drug that is a potent inhibitor of topoisomerase I. Which of the following is the most likely topoisomerase I inhibitor used for the treatment of this patient? A.Etoposide B.Doxorubicin C.Idarubicin D.Irinotecan E.Mitoxantrone Answer: D Irinotecan >> topoisomerase I; All others >> topoisomerase II

20 Relevant Slide No-35(Chapter9). A 45-year-old male has an insulinoma. Which of the following agents is the treatment of choice? a. Cyclophosphamide b. Carboplatin c. Vincristine d. Streptozocin e. Bleomycin

21 Relevant Slide No-37(Chapter9) A patient is being treated for non-Hodgkin's lymphoma with a nitrogen mustard-type antineoplastic agent developed gross hematuria. Which of the following agents could have been administered to prevent the onset of the patient's symptoms? A.Allopurinol B.Leucovorin C.C. Mesna D.D. Penicillamine E.E. Sodium thiosulfate

22 Relevant Slide No-38(Chapter9) 1.Which of the following drugs is most likely responsible for this nephrotoxicity? A. Bleomycin B. Cisplatin C. Cyclophosphamide D. Vinblastine E. Vincristine

23 Relevant Slide No-38(Chapter9) 2.A 45-year-old female treated for ovarian cancer develops difficulty hearing. Which of the following agents most likely caused these findings? a. Paclitaxel b. Doxorubicin c. Bleomycin d. 5-FU e. Cisplatin

24 Relevant Slide No-41(Chapter9) Binding to the enzyme dihydrofolate reductase is the mechanism of action for a. Procarbazine b. Paclitaxel c. Methotrexate d. Ifosfamide e. Cladribine

25 Relevant Slide No-42(Chapter9) 1.A nine-year-old boy is diagnosed with acute lymphoblastic leukemia. He is maintained on methotrexate. A recent platelet count is below normal, and a stool guaiac is 4+. Which of the following agents should be administered to counteract methotrexate toxicity? a. N-acetyl-L-cysteine b. Vitamin K c. Penicillamine d. Leucovorin e. Deferoxamine

26 Relevant Slide No-42(Chapter9) A 50-year-old female with rheumatoid arthritis has developed erosions in her wrist bones. Which of the following agents should be administered? [Remember another anticancer drug used in autoimmune disease] a. Allopurinol b. Asparaginase c. Methotrexate d. Streptozocin e. 6-MP

27 Relevant Slide No-44(Chapter9) 1. A young adult patient with acute granulocytic leukemia, treated with a combination of cytarabine and thioguanine, is no longer responsive to the therapy. The nonresponsiveness of the patient is thought to be due to thioguanine. What is the mechanism of resistance to thioguanine? a. Decreased uptake b. Increased efflux c. Increased alkaline phosphatase activity d. Increased production of trapping agents e. Increased DNA repair

28 Relevant Slide No-44(Chapter9). Identify drug to administer as pre- chemotherapeutic agent to reduce increased uric acid load caused by chemotherapeutic agent. Answer: Allopurinol

29 Relevant Slide No-45(Chapter9) 1.A 60-year-old female treated for breast cancer develops leukopenia and severe stomatitis and oral ulcerations. Which of the following agents most likely caused these findings? a. 5-FU b. Paclitaxel c. Cyclophosphamide d. Tamoxifen e. Carboplatin

30 Relevant Slide No-46(Chapter9) 1.which of the following agents would be suitable for the treatment of patient's lymphoma without further compromising his immune system? A. Busulfan B. Cisplatin C. Cyclophosphamide D. Paclitaxel E. Vincristine

31 Relevant Slide No-46(Chapter9) Which of the following is considered to be the effective mechanism of action of the vinca alkaloids? a. Inhibition of the function of microtubules b. Damage and prevention of repair of DNA c. Inhibition of DNA synthesis d. Inhibition of protein synthesis e. Inhibition of purine synthesis

32 Relevant Slide No-49(Chapter9) A 50-year-old female is treated with paclitaxel. Of the following, how is paclitaxel classified? a. An alkylating agent b. An antimetabolite c. A plant alkaloid d. An antibiotic e. A hormonal agent

33 Relevant Slide No-52(Chapter9) 1. A 58-year-old female develops a rather sudden onset of orthopnea, paroxysmal nocturnal dyspnea, and nocturia. On examination, the patient is tachycardic and both pulmonary rales and a third heart sound (S3) are noted. If the patient is receiving antineoplastic therapy for treatment of breast cancer, which of the following agents did she most likely receive? A. Bleomycin B. Carmustine C. Cisplatin D. Doxorubicin E. Methotrexate

34 Relevant Slide No-52(Chapter9). A 74-year-old woman with multiple myeloma is being treated with high doses of doxorubicin (Adriamycin). She has also received cyclophosphamide and prednisone recently. During his examination, the physician should check the patient for A. abdominal tenderness B. bladder distention C. limitation of movement D. papilledema E. pulmonary rales

35 Relevant Slide No-53(Chapter9) 1.A 32-year-old cancer patient, who has smoked two packs of cigarettes a day for 10 years, presents a decreased pulmonary function test. Physical examination and chest x-rays suggest preexisting pulmonary disease. Of the following drugs, which is best not prescribed? a. Vinblastine b. Doxorubicin c. Mithramycin d. Bleomycin e. Cisplatin

36 Relevant Slide No-62(Chapter9) A 52-year-old woman presents to her physician for a check- up. She is recovering from a wrist fracture after a fall. Dual energy x-ray absorptiometry of the hip had shown her to have osteoporosis. She became menopausal at age 50 and did not begin hormone replacement therapy because of a strong family history of breast cancer. She now fears a future hip fracture and would like to begin a bone loss prevention regime. Which of the following pharmaceutical agents is most appropriate for this patient? A. Calcitonin nasal spray B. Oral conjugated estrogen C. Raloxifene D. Tamoxifen E. Transdermal estradiol

37 Relevant Slide No-63(Chapter9). A 72-year-old man is diagnosed with prostate cancer. He declines surgical therapy, and is treated with flutamide. Which of the following is the mechanism of action of this drug? a.5 alpha-reductase inhibitor b.competitive antagonist at androgen receptors c.GnRH analog d.LH agonist e.Testosterone agonist

38 Relevant Slide No-64(Chapter9) 1.A 16-year-old male treated for acute lymphocytic leukemia develops severe lumbar and abdominal pain. His serum amylase is markedly elevated. Which of the following agents most likely caused these findings? a. 6-MP b. Asparaginase c. Doxorubicin d. Methotrexate e. Vincristine

39 Relevant Slide No-72(Chapter9) 1.A 58-year-old female develops a rather sudden onset of orthopnea, paroxysmal nocturnal dyspnea, and nocturia. On examination, the patient is tachycardic and both pulmonary rales and a third heart sound (S3) are noted. If the patient is receiving antineoplastic therapy for treatment of breast cancer, which of the following agents did she most likely receive? A. Bleomycin B. Carmustine C. Cisplatin D. Doxorubicin E. Methotrexate

40 Relevant Slide No-79(Chapter9) 1.A 30-year-old male with a two-year history of chronic renal failure requiring dialysis consents to transplantation. A donor kidney becomes available. He is given cyclosporine to prevent transplant rejection just before surgery. What is the most likely adverse effect of this drug? a. Bone marrow depression b. Nephrotoxicity c. Oral and GI ulceration d. Pancreatitis e. Seizures

41 Relevant Slide No-79(Chapter9) The most effective drug for immunosuppression of rejection of the allografted kidney is a. Azathioprine b. Cyclosporine c. 5-fluorouracil (5-FU) d. Cyclophosphamide e. Vincristine

42 Relevant Slide No-79(Chapter9) A 40-year-old female post–renal transplant has developed evidence of osteoporosis, most likely due to cyclosporine. Which of the following agents might replace cyclosporine? a. Allopurinol b. Asparaginase c. Methotrexate d. Streptozocin e. 6-MP f. Azathioprine

43 Relevant Slide No-79(Chapter9). A 45-year-old female has a bone marrow transplant for treatment of ovarian cancer. Cyclosporine is given as an immunosuppressant. What is the mechanism of action of cyclosporine? a. Direct destruction of proliferating lymphoid cells b. Inhibition of T cell response to cytokines c. Inhibition of folic acid metabolism d. Inhibition of factors that stimulate T cell growth e. Inhibition of enzymes that are related to purine metabolism

44 Relevant Slide No-81(Chapter9) 1.A 25-year-old female post–renal transplant shows signs of acute renal allograph rejection. Of the following agents, which should be administered? a. Interferon a b. Aldesleukin c. Muromonab-CD3 d. Sargramostim e. Filgrastim


Download ppt "Anticancer & Immunosupressants Q.. Questions 1-3. For each of the following statements, select the most appropriate drug which is BEST characterized by."

Similar presentations


Ads by Google